next up previous
Next: Υπόδειξη Up: Ασκήσεις Previous: Λύση


'Ασκηση 20

Δείξτε ότι

\begin{displaymath}(\cos \theta)^p \leq \cos(p\theta),\end{displaymath}

για $0 \leq \theta \leq \frac{\pi}{2}$ και $0 < p < 1.$ Υπόδειξη Λύση





Antonis Tsolomitis
1999-11-11